Proving the limit for the number e

  • Thread starter Xcron
  • Start date
  • Tags
    Limit
In summary, we can use L'Hospital's Rule to prove that the limit of (1+x/n)^n as n approaches infinity is e^x for any x>0. We can also use the binomial theorem and the definition of e^x in terms of its Taylor series to prove this limit.
  • #1
Xcron
22
0
Ok, the problem says:

Show that [tex]\lim_{n\rightarrow\infty} (1+\frac{x}{n})^n = e^x[/tex] for any [tex]x>0[/tex].I thought that I could say that y = 1+x/n...and then use the natural logarithm to narrow it down to [tex]\ln y=n\ln(\frac{x}{n})[/tex] ... I should be getting [tex]x[/tex] so that when I take it back into the original limit, I would have [tex]e^x[/tex] but I can't seem to make it that way..
 
Physics news on Phys.org
  • #2
Did you mean to make y a function of both x and n? (So that it's value is not a constant as n goes to infinity)
 
  • #3
Xcron said:
Ok, the problem says:

Show that [tex]\lim_{n\rightarrow\infty} (1+\frac{x}{n})^n = e^x[/tex] for any [tex]x>0[/tex].


I thought that I could say that y = 1+x/n...and then use the natural logarithm to narrow it down to [tex]\ln y=n\ln(\frac{x}{n})[/tex] ... I should be getting [tex]x[/tex] so that when I take it back into the original limit, I would have [tex]e^x[/tex] but I can't seem to make it that way..

That [tex]\lim_{n\rightarrow\infty} (1+\frac{x}{n})^n = e^x[/tex] may be proven as follows:

[Is this for a real analysis class, or a calculus class? If this is for a calculus class, ignore this:]

by the binomial theorem,

[tex](1+\frac{x}{n})^n = \sum_{k=0}^{n} \frac{n!}{k!(n-k)!}\left( \frac{x}{n}\right) ^{k} = \sum_{k=0}^{n} \frac{n!}{(n-k)!}\frac{1}{n^{k}} \frac{x^k}{k!}\rightarrow \sum_{k=0}^{\infty} \frac{x^k}{k!}=:e^{x}\mbox{ as }n\rightarrow\infty[/tex]

since [tex] \frac{n!}{(n-k)!}\frac{1}{n^{k}} \rightarrow 1 \mbox{ as }n\rightarrow\infty[/tex]

although there are some uniform convergence issues to be handled when taking the limit of the above sum...
 
Last edited:
  • #4
Xcron said:
Ok, the problem says:

Show that [tex]\lim_{n\rightarrow\infty} (1+\frac{x}{n})^n = e^x[/tex] for any [tex]x>0[/tex].


I thought that I could say that y = 1+x/n...and then use the natural logarithm to narrow it down to [tex]\ln y=n\ln(\frac{x}{n})[/tex] ... I should be getting [tex]x[/tex] so that when I take it back into the original limit, I would have [tex]e^x[/tex] but I can't seem to make it that way..

Well taking the natural log defeats the purpose of the original proof, because the natural log depends on the value of e, in other words your using the number to find it so it has no meaning at all because you had to have already known it in the first place.
 
  • #5
Xcron, do please post what definitions you have been given for e and for ex.
 
  • #6
Sorry, I didn't completely specify the directions of the problem. I am required to use L'Hospital's Rule to solve this limit (this is a Calculus class). I was going to us ln y in order to simplify the limit a bit.

The definition for the number e is standard I guess...the base of the natural logarithm..
 
  • #7
To prove [tex]\lim_{n\rightarrow\infty} (1+\frac{x}{n})^n = e^x[/tex]

Let [tex]y=\lim_{n\rightarrow\infty} \left( 1+\frac{x}{n}\right) ^n[/tex].
Then [tex]\ln y=\lim_{n\rightarrow\infty} n\ln \left( 1+\frac{x}{n}\right) =\lim_{n\rightarrow\infty} \frac{\ln \left( 1+\frac{x}{n}\right)}{\frac{1}{n}} = \lim_{n\rightarrow\infty} \frac{\frac{1}{1+\frac{x}{n}}\left( -\frac{x}{n^2}\right) }{-\frac{1}{n^2}} = \lim_{n\rightarrow\infty} \frac{x}{1+\frac{x}{n}}=x[/tex]

hence [tex]y=e^x[/tex].
 
Last edited:
  • #8
Could you please explain the mechanics of the two variables?

I'm not sure how to go about thinking/reasoning the presence of both..
 
  • #9
x is a constant, n is "the" variable for l'Hospital's Rule (tripped me up the firsty run through of the problem too.)
 
  • #10
Ahhhh, that was my final guess that I was making as to how they would work. It seemed like we weren't supposed to touch/work with x, and thus the main variable that we should be concerned with is n (since that was the target of the limit).
 
  • #11
I like the rigourous version offered by benorin in post #3, seems like the best proof, and it uses e^x defined in terms of its Taylor series. I understood most of it. The only thing is, I can't remember why that expression ---> 1 as n ---> infinity in the last step.

Obviously though, post #7 is exactly what the instructor wanted.

I'm not sure if you guys will approve of this "proof", but I know another way of showing it that depends on defining e^x as the exponential function whose derivative is the function itself. So we want:

[tex] \frac{d}{dx}(e^x) = e^x [/tex]

[tex] \lim_{h \to 0} \frac{e^{x+h} - e^x}{h} = e^x [/tex]

[tex] \lim_{h \to 0} \frac{e^{x}e^h - e^x}{h} = e^x [/tex]

[tex] \lim_{h \to 0} \frac{e^h - 1}{h}e^x = e^x [/tex]

This requires that the number e is a base of the exponential function that satisfies the condition:

[tex] \lim_{h \to 0} \frac{e^h - 1}{h} = 1 [/tex]

[tex] \lim_{h \to 0} e^h - 1 = \lim_{h \to 0} h [/tex]

[tex] \lim_{h \to 0} e^h = \lim_{h \to 0} h + 1 [/tex]

[tex] e = \lim_{h \to 0} (h + 1)^{\frac{1}{h}} [/tex]

Let [itex] n \equiv 1/h [/itex]

[tex] e = \lim_{n \to \infty} (1 + \frac{1}{n})^{n} [/tex]

change variables (x doesn't depend on n, can be considered a constant here)

[tex] e = \lim_{n/x \to \infty} (1 + \frac{x}{n})^{n/x} [/tex]

[tex] = \lim_{n \to \infty} (1 + \frac{x}{n})^{n/x} [/tex]

It follows that:

[tex] e^x = \lim_{n \to \infty} (1 + \frac{x}{n})^{(xn/x)} [/tex]

[tex] = \lim_{n \to \infty} (1 + \frac{x}{n})^{(n)} [/tex]
 
  • #12
This is not rigorous, but it works...

cepheid said:
I like the rigourous version offered by benorin in post #3, seems like the best proof, and it uses e^x defined in terms of its Taylor series. I understood most of it. The only thing is, I can't remember why that expression ---> 1 as n ---> infinity in the last step.

This is not rigorous, but it works...

[tex] \frac{n!}{(n-k)!} = \frac{n(n-1)(n-2)\cdots (n-(k-1))(n-k)!}{(n-k)!} =n(n-1)(n-2)\cdots (n-(k-1))[/tex]

counting the number of terms in the above product: it goes n-0 through n-(k-1) so there are (k-1)-0+1 = k terms so we know that the leading term will be nk when the product is expanded, and hence

[tex] \frac{n!}{(n-k)!} = n^k + \mbox{ some polynomial of degree k-1 in }n[/tex]

or rather

[tex] \frac{n!}{(n-k)!}\frac{1}{n^{k}} \rightarrow 1 \mbox{ as }n\rightarrow\infty[/tex]

-Ben
 

1. What is the limit for the number e?

The limit for the number e is an irrational number, approximately equal to 2.71828. It is also known as Euler's number and is a fundamental constant in mathematics.

2. How is the limit for the number e calculated?

The limit for the number e is calculated using the infinite series e = 1 + 1/1! + 1/2! + 1/3! + 1/4! + ... + 1/n! as n approaches infinity. This series is also known as the Maclaurin series for e.

3. What is the significance of the limit for the number e?

The limit for the number e is significant because it appears in many areas of mathematics and science, including calculus, complex numbers, and probability. It also has many real-world applications, such as compound interest and exponential growth.

4. Who discovered the limit for the number e?

The limit for the number e was discovered by Swiss mathematician Leonhard Euler in the 18th century. However, the number itself was known earlier by mathematicians such as John Napier and Jacob Bernoulli.

5. Can the limit for the number e be proven?

Yes, the limit for the number e can be proven using various mathematical techniques, such as the Maclaurin series and the concept of limits in calculus. It is a well-established mathematical constant and has been proven to be an irrational number.

Similar threads

  • Calculus and Beyond Homework Help
Replies
13
Views
635
  • Calculus and Beyond Homework Help
Replies
17
Views
485
  • Calculus and Beyond Homework Help
Replies
8
Views
588
  • Calculus and Beyond Homework Help
Replies
6
Views
869
  • Calculus and Beyond Homework Help
Replies
6
Views
418
  • Calculus and Beyond Homework Help
Replies
10
Views
792
  • Calculus and Beyond Homework Help
Replies
2
Views
658
  • Calculus and Beyond Homework Help
Replies
13
Views
2K
  • Calculus and Beyond Homework Help
Replies
20
Views
1K
  • Calculus and Beyond Homework Help
Replies
9
Views
1K
Back
Top